4
$\begingroup$

Let $C$ be a curve in $\mathbb{P}^3$, possibly non-reduced. Assume, there exists a smooth surface in $\mathbb{P}^3$ containing $C$. Is it true that for $d \gg 0$, a generic element of $I_d(C)$ defines a smooth surface in $\mathbb{P}^3$?

$\endgroup$

2 Answers 2

5
$\begingroup$

I hope this is not a homework exercise, but I do not recall this from the standard textbooks.

This is already false for planar double lines. Let $\mathbb{P}^3$ have homogeneous coordinates $[T_0,T_1,T_2,T_3]$. Let $S$ be the plane $Z(T_3)$. Let $C$ be the curve $Z(T_2^2,T_3)$ with induced reduced curve $L=Z(T_2,T_3)$. Let $R$ be a smooth hypersurface of degree $d$ in $\mathbb{P}^3$ that contains $L$. Since $C$ is a local complete intersection scheme, if $R$ contains $C$, then $C$ is a Cartier divisor on $R$. Thus $[C]$ equals $2[L]$ as an effective Cartier divisor on $R$. In particular, the kernel $\mathcal{J}$ of the reduction homomorphism, $$ \mathcal{O}_C \to \mathcal{O}_L, $$ equals $\mathcal{O}_R(-L)|_L$, i.e., the dual of the normal sheaf of $L$ in $R$ (the conormal sheaf).

Considering the special case when $R$ equals $S$, we compute that $\mathcal{J}$ is isomorphic to $\mathcal{O}_L(-1)$. On the other hand, by adjunction, Chern class computations, etc., for general $R$ of degree $d$ that contains $L$, $\mathcal{O}_R(-L)|_L$ is isomorphic to $\mathcal{O}_L(2-d)$. Thus, if $R$ contains $C$, then $d$ must equal $1$.

$\endgroup$
2
$\begingroup$

EDIT. The first version of this answer claimed that the result was true for any $C$, but it was uncorrect as remarked by J. Starr. In fact, Jason's answer shows that the result may be false for a non-reduced $C$.

However, the claim is true when $C$ is reduced; let me give a sketch of the proof.

Since $\mathcal{O}_{\mathbb{P}^3}(1)$ is ample, there exists $d_0 \in \mathbb{N}$ such that for any $d \geq d_0$ the sheaf $\mathcal{I}_C(d)$ is generated by its global sections. Hence the base locus of $|\mathcal{I}_C(d)|$ consists of the curve $C$ only. In particular, the linear system $|\mathcal{I}_C(d)|$ has not fixed components, so Bertini theorem implies that

the general element of the linear system $|\mathcal{I}_C(d)|$ is smooth outside $C$. $(*)$

Let now $S$ be a smooth surface of degree $n$ containing $C$ (we may assume $d_0 > n$) and consider the elements in $|\mathcal{I}_C(d)|$ of the form $$X=S +H_1+H_2+ \cdots + H_{d-n},$$ where the $H_i$ are hyperplanes.

If $p$ is any point of $C$ and we choose as $H_i$ hyperplanes not containing $p$, then $X$ is a surface of degree $d$ which is smooth at $p$.

Since $p \in C$ is arbitrary, by using $(*)$ it follows that the general element of $|\mathcal{I}_C(d)|$ is smooth everywhere for $d \geq d_0$.

$\endgroup$
12
  • 3
    $\begingroup$ Your argument is incorrect. Let $S$ be a linear plane, and let $C$ be a planar curve of very high degree. By Bezout's theorem, every quadric surface that contains $C$ must also contain the plane $S$. Bertini's theorem only says that the general member of the linear system is smooth away from the base locus. However, $S$ is in the base locus of the system of surfaces obtained as $S+H$ by varying $H$. $\endgroup$ Apr 2, 2013 at 17:30
  • $\begingroup$ Right, this argument must be fixed. Unfortunately I have no time now, I will do it later. Thank you! $\endgroup$ Apr 2, 2013 at 18:15
  • 1
    $\begingroup$ @Olivier: it seems to me that the argument proving that the base locus of $I_C(d)$ consists of $C$ only (for $d >>0$) assumes that $C$ is considered with its reduced structure, and that it cannot be carried out if instead $C$ has some nilpotent structure (as Jason's example shows). I'm missing something? $\endgroup$ Apr 3, 2013 at 8:52
  • 1
    $\begingroup$ I don't follow either. In Jason's example, the singularities of $R$ lie on the base locus (and for every $d>1$, the base locus of $\mathcal{I}_C(d)$ is just $C$). But these singularities move with $R$. That is, for every $p\in C$ there is indeed a surface smooth at $p$ and containing $C$, but it must be singular at some other point $p\in C$. $\endgroup$
    – quim
    Apr 3, 2013 at 11:47
  • 1
    $\begingroup$ Rather, the other point must be $q\in C$ ;) $\endgroup$
    – quim
    Apr 3, 2013 at 11:48

Your Answer

By clicking “Post Your Answer”, you agree to our terms of service and acknowledge you have read our privacy policy.

Not the answer you're looking for? Browse other questions tagged or ask your own question.